Probability (altered die problem)

  • Thread starter Thread starter bobsz
  • Start date Start date
  • Tags Tags
    Probability
Click For Summary
The discussion revolves around calculating the probability of selecting an altered die after rolling two "1s." The original calculation mistakenly assessed the probability of outcomes and intersections. The correct approach indicates that the probability of rolling two "1s" with the altered die is 1/6, while the total outcomes need to be recalculated to reflect the correct probabilities. The consensus is that the probability of selecting the altered die is indeed higher than 1/3, highlighting the importance of accurate probability assessment in such problems. Overall, the thread emphasizes the need for careful consideration of outcomes when solving probability problems.
bobsz
Messages
3
Reaction score
0
Hello everyone,

I need some help with the following prob:

A bag contains 3 dice, 2 fair and 1 altered with all odd numbers replace with "1". One die is randomly selected and rolled independently twice. If the outcomes of both rolls were "1" and "1", what is the prob that the selected die is the altered die?
Here's what I've done:

Let A= Altered die and B=outcome is 1

then P(A|B) = P(A∩B)/ P(B)
P(A∩B) = 1/12 ( outcome of 3 11's)
P(B)= 5/9 ( total outcome of 1's)

therefore P(A|B) = (1/12)/(5/9) = 3/20

Is this correct? Is there another way to approach this problem?

Thanks!
 
Physics news on Phys.org
First, it should be obvious that the odds of the altered die being picked is greater than 1/3.

In your attempt, the P(B) isn't 5/9, since you can only get 5 1s out of 18 total numbers. And the probability of it being the altered die and the outcome is one is actually 1/3*1/2, which is 1/6, not 1/12. That gives your method a much better answer.
 
Question: A clock's minute hand has length 4 and its hour hand has length 3. What is the distance between the tips at the moment when it is increasing most rapidly?(Putnam Exam Question) Answer: Making assumption that both the hands moves at constant angular velocities, the answer is ## \sqrt{7} .## But don't you think this assumption is somewhat doubtful and wrong?

Similar threads

  • · Replies 1 ·
Replies
1
Views
1K
  • · Replies 16 ·
Replies
16
Views
2K
  • · Replies 6 ·
Replies
6
Views
2K
  • · Replies 2 ·
Replies
2
Views
2K
  • · Replies 6 ·
Replies
6
Views
2K
  • · Replies 4 ·
Replies
4
Views
2K
Replies
3
Views
1K
  • · Replies 13 ·
Replies
13
Views
2K
  • · Replies 5 ·
Replies
5
Views
1K
  • · Replies 3 ·
Replies
3
Views
2K